3
$\begingroup$

Applying the Müntz–Szász theorem on $[0,1]$ repeatedly, we can represent $$ x= \sum_{n\geq 2} c_n x^n $$ as a uniformly convergent series (edit: only over some subsequence, see edits below) on $[0,1]$ of higher powers $x^n$ for $n\geq 2$. What can one say about the coefficients? Is there an explicit choice of $c_n$?

Edit: Comments below suggest that this is not possible. What is wrong with the following argument? Take $\epsilon>0$ and approximate $x$ by a finite combination of higher powers and a constant uniformly with an error $\epsilon/2.$ Plugging $x=0$ we see that the constant is smaller than $\epsilon/2$ so dropping it we get an approximation up to $\epsilon$ by a finite sum $\sum_{n=2}^{N_1} c_n x^n.$ Next, consider $x-\sum_{n=2}^{N_1} c_n x^n$ and approximate it by a linear combination $\sum_{n=N_1+1}^{N_2} c_n x^n$ up to an error $\epsilon/2.$ This gives us an $\epsilon/2$-approximation $\sum_{n=1}^{N_2}c_n x^n.$ Continue this construction repeatedly.

Edit II: Theorem holds for $a=0$ if we include constants, but Robert Israel's comment below contained the main point: the series only converges over some subsequence $(N_k)_{k\geq 1}$ as in the above construction. Let me rephrase the question accordingly:

Is there anything interesting one can say about $c_n$? Can one choose the subsequence and $c_n$ in a way that $(c_n)\in\ell^p$ for some $p$, or uniformly bounded?

$\endgroup$
4
  • $\begingroup$ can you perhaps shed some light onto how this might work near $x=0$? $\endgroup$ Feb 7, 2017 at 15:13
  • 3
    $\begingroup$ @CarloBeenakker It won't work - the theorem only holds on intervals $[a,b]$ with $a>0$. $\endgroup$
    – Dirk
    Feb 7, 2017 at 15:16
  • 3
    $\begingroup$ You can't get a convergent series, either: only some subsequence of the partial sums converges. $\endgroup$ Feb 7, 2017 at 15:24
  • 1
    $\begingroup$ As has been pointed out, a convergent series would define a holomorphic function near $x=0$, which cannot have two distinct power series representations. The problem with your argument is that the $c_n$'s will depend on $\epsilon$, so you're not constructing a series at all but a more general sequence of polynomials. $\endgroup$ Feb 7, 2017 at 17:34

2 Answers 2

4
$\begingroup$

It can't be in $\ell^p$ or bounded, in fact you can't have $|c_n| = O(t^{-n})$ for any $t > a$ where the subsequence converges on $[a,1]$. This is because if $|c_n| = O(t^{-n})$, $\sum_n c_n z^n$ is analytic in $|z|<t$, and by uniqueness...

$\endgroup$
2
$\begingroup$

[Sorry -- too long for a comment]. It has already been pointed out that if you want 0 in your domain then you'll have to use constant functions too. But even then the theorem does not say what you seem to think it says: it only says that we can approximate $f(x)=x$ arbitrarily well by such a power series -- and as the approximation gets better and better the terms might jump around like crazy. In particular the coefficients are not really well-defined. As one way of seeing this, take one expansion which is very close to $x$ and let's assume the coefficient of $x^7$ is non-zero. Then remove 7 from your set of allowable powers of $x$ and the theorem again says that one can approximate $x$ arbitrarily well; this time the coefficient of $x^7$ is forced to be zero.

$\endgroup$
4
  • $\begingroup$ What is wrong with the argument I sketched in my edit? $\endgroup$ Feb 7, 2017 at 15:40
  • 1
    $\begingroup$ @C.Eratosthene As Dirk points out, the theorem holds only on intervals $[a,b]$ with $a>0$. Your arguments actually show why this condition is necessary. $\endgroup$ Feb 7, 2017 at 16:02
  • $\begingroup$ Robert Israel's objection is the point -- your resulting infinite series is not guaranteed to converge. You only know that it's well-behaved if you sum up to $N_1$, $N_2$ and so on. The intermediate sums will presumably be all over the place. $\endgroup$ Feb 7, 2017 at 16:37
  • 1
    $\begingroup$ In fact one can I guess see this by fixing $N$ and then looking at behaviour for $x=\delta$ chosen so small that $x$ is far larger than $\sum_{i=1}^Nc_ix^i$. This then shows that some term beyond the $N$th term has to be gigantic. $\endgroup$ Feb 7, 2017 at 16:40

Your Answer

By clicking “Post Your Answer”, you agree to our terms of service and acknowledge you have read our privacy policy.

Not the answer you're looking for? Browse other questions tagged or ask your own question.